Stay ahead of learning milestones! Enroll in a class over the summer!

G
Topic
First Poster
Last Poster
Problem 1
SpectralS   146
N an hour ago by YaoAOPS
Given triangle $ABC$ the point $J$ is the centre of the excircle opposite the vertex $A.$ This excircle is tangent to the side $BC$ at $M$, and to the lines $AB$ and $AC$ at $K$ and $L$, respectively. The lines $LM$ and $BJ$ meet at $F$, and the lines $KM$ and $CJ$ meet at $G.$ Let $S$ be the point of intersection of the lines $AF$ and $BC$, and let $T$ be the point of intersection of the lines $AG$ and $BC.$ Prove that $M$ is the midpoint of $ST.$

(The excircle of $ABC$ opposite the vertex $A$ is the circle that is tangent to the line segment $BC$, to the ray $AB$ beyond $B$, and to the ray $AC$ beyond $C$.)

Proposed by Evangelos Psychas, Greece
146 replies
SpectralS
Jul 10, 2012
YaoAOPS
an hour ago
Bounding number of solutions for floor function equation
Ciobi_   1
N an hour ago by sarjinius
Source: Romania NMO 2025 9.3
Let $n \geq 2$ be a positive integer. Consider the following equation: \[ \{x\}+\{2x\}+ \dots + \{nx\} = \lfloor x \rfloor + \lfloor 2x \rfloor + \dots + \lfloor 2nx \rfloor\]a) For $n=2$, solve the given equation in $\mathbb{R}$.
b) Prove that, for any $n \geq 2$, the equation has at most $2$ real solutions.
1 reply
Ciobi_
Apr 2, 2025
sarjinius
an hour ago
nice system of equations
outback   4
N 2 hours ago by Raj_singh1432
Solve in positive numbers the system

$ x_1+\frac{1}{x_2}=4, x_2+\frac{1}{x_3}=1, x_3+\frac{1}{x_4}=4, ..., x_{99}+\frac{1}{x_{100}}=4, x_{100}+\frac{1}{x_1}=1$
4 replies
outback
Oct 8, 2008
Raj_singh1432
2 hours ago
Two circles, a tangent line and a parallel
Valentin Vornicu   103
N 2 hours ago by zuat.e
Source: IMO 2000, Problem 1, IMO Shortlist 2000, G2
Two circles $ G_1$ and $ G_2$ intersect at two points $ M$ and $ N$. Let $ AB$ be the line tangent to these circles at $ A$ and $ B$, respectively, so that $ M$ lies closer to $ AB$ than $ N$. Let $ CD$ be the line parallel to $ AB$ and passing through the point $ M$, with $ C$ on $ G_1$ and $ D$ on $ G_2$. Lines $ AC$ and $ BD$ meet at $ E$; lines $ AN$ and $ CD$ meet at $ P$; lines $ BN$ and $ CD$ meet at $ Q$. Show that $ EP = EQ$.
103 replies
Valentin Vornicu
Oct 24, 2005
zuat.e
2 hours ago
Inequalities
idomybest   3
N 3 hours ago by damyan
Source: The Interesting Around Technical Analysis Three Variable Inequalities
The problem is in the attachment below.
3 replies
idomybest
Oct 15, 2021
damyan
3 hours ago
Function on positive integers with two inputs
Assassino9931   2
N 3 hours ago by Assassino9931
Source: Bulgaria Winter Competition 2025 Problem 10.4
The function $f: \mathbb{Z}_{>0} \times \mathbb{Z}_{>0} \to \mathbb{Z}_{>0}$ is such that $f(a,b) + f(b,c) = f(ac, b^2) + 1$ for any positive integers $a,b,c$. Assume there exists a positive integer $n$ such that $f(n, m) \leq f(n, m + 1)$ for all positive integers $m$. Determine all possible values of $f(2025, 2025)$.
2 replies
Assassino9931
Jan 27, 2025
Assassino9931
3 hours ago
Normal but good inequality
giangtruong13   4
N 3 hours ago by IceyCold
Source: From a province
Let $a,b,c> 0$ satisfy that $a+b+c=3abc$. Prove that: $$\sum_{cyc} \frac{ab}{3c+ab+abc} \geq \frac{3}{5} $$
4 replies
giangtruong13
Mar 31, 2025
IceyCold
3 hours ago
Tiling rectangle with smaller rectangles.
MarkBcc168   61
N 3 hours ago by YaoAOPS
Source: IMO Shortlist 2017 C1
A rectangle $\mathcal{R}$ with odd integer side lengths is divided into small rectangles with integer side lengths. Prove that there is at least one among the small rectangles whose distances from the four sides of $\mathcal{R}$ are either all odd or all even.

Proposed by Jeck Lim, Singapore
61 replies
MarkBcc168
Jul 10, 2018
YaoAOPS
3 hours ago
A magician has one hundred cards numbered 1 to 100
Valentin Vornicu   49
N 3 hours ago by YaoAOPS
Source: IMO 2000, Problem 4, IMO Shortlist 2000, C1
A magician has one hundred cards numbered 1 to 100. He puts them into three boxes, a red one, a white one and a blue one, so that each box contains at least one card. A member of the audience draws two cards from two different boxes and announces the sum of numbers on those cards. Given this information, the magician locates the box from which no card has been drawn.

How many ways are there to put the cards in the three boxes so that the trick works?
49 replies
Valentin Vornicu
Oct 24, 2005
YaoAOPS
3 hours ago
Nice inequality
sqing   2
N 3 hours ago by Seungjun_Lee
Source: WYX
Let $a_1,a_2,\cdots,a_n  (n\ge 2)$ be real numbers . Prove that : There exist positive integer $k\in \{1,2,\cdots,n\}$ such that $$\sum_{i=1}^{n}\{kx_i\}(1-\{kx_i\})<\frac{n-1}{6}.$$Where $\{x\}=x-\left \lfloor x \right \rfloor.$
2 replies
sqing
Apr 24, 2019
Seungjun_Lee
3 hours ago
Concurrency
Dadgarnia   27
N 3 hours ago by zuat.e
Source: Iranian TST 2020, second exam day 2, problem 4
Let $ABC$ be an isosceles triangle ($AB=AC$) with incenter $I$. Circle $\omega$ passes through $C$ and $I$ and is tangent to $AI$. $\omega$ intersects $AC$ and circumcircle of $ABC$ at $Q$ and $D$, respectively. Let $M$ be the midpoint of $AB$ and $N$ be the midpoint of $CQ$. Prove that $AD$, $MN$ and $BC$ are concurrent.

Proposed by Alireza Dadgarnia
27 replies
Dadgarnia
Mar 12, 2020
zuat.e
3 hours ago
Right angles
USJL   10
N Apr 13, 2025 by bin_sherlo
Source: 2018 Taiwan TST Round 3
Let $I$ be the incenter of triangle $ABC$, and $\ell$ be the perpendicular bisector of $AI$. Suppose that $P$ is on the circumcircle of triangle $ABC$, and line $AP$ and $\ell$ intersect at point $Q$. Point $R$ is on $\ell$ such that $\angle IPR = 90^{\circ}$.Suppose that line $IQ$ and the midsegment of $ABC$ that is parallel to $BC$ intersect at $M$. Show that $\angle AMR = 90^{\circ}$

(Note: In a triangle, a line connecting two midpoints is called a midsegment.)
10 replies
USJL
Apr 2, 2020
bin_sherlo
Apr 13, 2025
Right angles
G H J
G H BBookmark kLocked kLocked NReply
Source: 2018 Taiwan TST Round 3
The post below has been deleted. Click to close.
This post has been deleted. Click here to see post.
USJL
535 posts
#1 • 2 Y
Y by Infinityfun, MathLuis
Let $I$ be the incenter of triangle $ABC$, and $\ell$ be the perpendicular bisector of $AI$. Suppose that $P$ is on the circumcircle of triangle $ABC$, and line $AP$ and $\ell$ intersect at point $Q$. Point $R$ is on $\ell$ such that $\angle IPR = 90^{\circ}$.Suppose that line $IQ$ and the midsegment of $ABC$ that is parallel to $BC$ intersect at $M$. Show that $\angle AMR = 90^{\circ}$

(Note: In a triangle, a line connecting two midpoints is called a midsegment.)
Z K Y
The post below has been deleted. Click to close.
This post has been deleted. Click here to see post.
parmenides51
30630 posts
#2
Y by
solved by enhanced as problem 32 here
SOLUTION TO PROBLEM:32
Z K Y
The post below has been deleted. Click to close.
This post has been deleted. Click here to see post.
FISHMJ25
293 posts
#3
Y by
Okay here is moving points solutions that enables us to work with complex numbers. First move $P$ on the circle $(ABC)$. Then
$$P\mapsto Q\mapsto M$$is obviously projective so $M$ moves with deg $1$. Next $R$ moves with deg $4$. Now the claim is equivalent with
showing that $ \infty_{ \perp AK}$ $K$ and $R$ are collinear. So we need to check problem statement in $1+1+4+1=7$ cases. Let $D,E$ and $F$ be midpoints of arcs $A$, $B$ and $C$ (that are opposite to $A$, $B$ and $C$). Claim is well known for $P=E,F$, and is trivial for $P=D$.
Now something harder, let $P=A$. Then $Q=AA \cap EF$ and $R=\infty_{EF}$ so we need to show $IQ||BC$. Here we employ complex numbers.
$a=x^2,b=y^2,c=z^2$ Calculating $Q$ we get $q=x^2\frac{x(y+z)+2yz}{yz-x^2}$. Its left to show $\frac{q-I}{\bar q - \bar I}=-bc$.
$$q+I=x^2\frac{x(y+z)+2yz}{yz-x^2}+\sum xy$$$$yz\frac{(x+y)(x+z)}{yz-x^2}$$And now it is easily seen that we are done in this case.
Next one is interesting: take $P$ on $(ABC)$ s.t. $<API=90$. Again bash it but little bit smarter this time. First calculate $t=ef\cap uw$ where $U$ and $W$ are intersections of $KL$ with $(ABC)$. We easily see $u+w=\frac{1}{2}(a+b+c+\frac{bc}{a})$ and $uw=bc$. Now we can use intersection formula to get $$t=\frac{x^2\sum xy+y^2z^2+2xyz(y+z)}{2(x^2-yz)}$$Notice that $\frac{p-a}{\bar p - \bar a}=-\frac{p-(-a)}{\bar t - \bar (-a)}$
We have avoided calculating $P$ (its not that bad either, but this is nice trick). Next it is sufficint to show $$\frac{t-a}{\bar t - \bar a}=\frac{t-a}{\bar t - \bar a}=-\frac{p-(-a)}{\bar t -  (-\bar a)}$$Which is equivalent with $\frac{t-a}{I-(-a))} \in R$ But $$t-a=\frac{(x+y)^2(x+z)^2}{2(x^2-yz}$$And again as before its easily seen that we are done. And now the hardest case $P=C$ (we symmetricaly get $B$). Then calculating we get $r=\frac{x(z^2+xy)+xz(y+z)}{x-z}$ Via angle chasing we can see $IM||AB$ then its not hard to see $$m=\frac{2xyz \sum x+2z^2 \sum xy+ (x^2+y^2)(x^2+z^2) }{x^2-z^2}$$Next just calculate $\frac{m-a}{m-r} \in iR$ $$m-a=\frac{(x+z)(x+y)(\sum xy-x^2+2z^2)}{x^2-z^2}$$$$m-r=\frac{(y-x)(x+z)(\sum xy-x^2+2z^2)}{x^2-z^2}$$Now its trivial to check that we are done. Just noticed sythetic for $P=C$. Note that if $KL \cap RC=S$ then $AS\perp RC$ and claim follows easily.
Note: My favorite is case $5$ (where $<API=90$).
This post has been edited 1 time. Last edited by FISHMJ25, Sep 30, 2020, 11:47 AM
Z K Y
The post below has been deleted. Click to close.
This post has been deleted. Click here to see post.
Cindy.tw
54 posts
#4
Y by
Nice problem!

Let $D$ is the intersection of $BC$ and $AM$, we prove that $\triangle AIP \sim \triangle IDP$. Let $t$ be a line passing through $D$ tangent to incircle, by second Fontene's theorem, $I$ lies on the Newton's line of it, so $IM$ is the Newton's line of quadrangle $\mathcal{Q}(BC, AC, AB, t)$. It's well-known that Miquel point and infinite of Newton's line are isogonal conjugate, hence $P$ is the Miquel point of $\mathcal{Q}(BC, AC, AB, t)$,so $\triangle AIP \sim \triangle IDP$. $J$ is the point symmetric of $I$ WRT $P$, then $A, I, D, J$ are harmonic quadrilateral, so $R$ is the circumcenter of $\odot(AIDJ)$, $RM$ is perpendicular to $AM$, we're done.
Z K Y
The post below has been deleted. Click to close.
This post has been deleted. Click here to see post.
KST2003
173 posts
#5 • 2 Y
Y by CrazyMathMan, iamnotgentle
Let $F=\overline{AI}\cap\overline{BC}$, $L=\overline{AI}\cap(ABC)$, and let $X$ and $N$ be the midpoints of segments $AF$ and $AI$. Let $M'$ and $X'$ be the reflection of $M$ and $X$ over $l$, and let $O$ be the circumcenter of $\triangle ABC$. Notice that $M'$ lies on $\overline{AQ}$. Since $\angle AMR = \angle IM'R$, it suffices to show that $M'$ lies on $(INRP)$. Let $k$ be the reflection of $A$-midline over $l$; this passes through $M'$ and $X$.

Claim: $k$ is the image of $(ABC)$ under inversion at $A$ with radius $\sqrt{AN \cdot AI}$.

Proof. Since $\overline{AI}$ and $l$ are perpendicular to each other, $k$ is parallel to the line obtained by reflecting the $A$-midline over $\overline{AI}$. i.e. $k$ is anti-parallel to $\overline{BC}$ in $\angle A$, and hence it is perpendicular to $\overline{AO}$. Therefore, to show the claim, we just need to show that $X'$ is the inverse of $L$ under inversion, or
\[AX' \cdot AL = AN \cdot AI \Longleftrightarrow \frac{AX'}{AN} = \frac{AI}{AL}\]Just notice that
\[\frac{AN}{NX'} = \frac{AN}{NX} = \frac{AI}{IF} = \frac{AC}{CF} = \frac{AL}{LB} = \frac{AL}{LI}\]so $\{A, X', N\} \sim \{A, I, L\}$, and the claim follows. $\square$

As $AM' \cdot AP = AX \cdot AL = AN \cdot AI$, it follows that $M'$ lies on $(INP)$ so we are done. $\blacksquare$
[asy]
defaultpen(fontsize(10pt));
size(12cm);
pen mydash = linetype(new real[] {5,5});
pair A = dir(120);
pair B = dir(225);
pair C = dir(315);
pair I = incenter(A,B,C);
pair F = extension(A,I,B,C);
pair O = circumcenter(A,B,C);
pair L = 2*foot(O,A,I)-A;
pair X = midpoint(A--F);
pair N = midpoint(A--I);
pair X1 = 2*N-X;
pair S = midpoint(A--C);
pair T = midpoint(A--B);
pair P = dir(238);
pair D = 2*foot(O,B,I)-B;
pair E = 2*foot(O,C,I)-C;
pair Q = extension(A,P,D,E);
pair M = extension(Q,I,S,T);
pair R = extension(P,rotate(90,P)*I,D,E);
pair M1 = 2*foot(M,D,E)-M;
real s = 0.9;
pair K1 = s*unit(M1-X1) + extension(M1,X1,A,O);
pair K2 = s*unit(X1-M1) + extension(M1,X1,A,O);
draw(K1--K2, royalblue+1);
draw(A--B--C--cycle, black+1);
draw(B--D, mydash);
draw(C--E, mydash);
draw(R--D);
draw(A--L);
draw(A--P);
draw(P--R);
draw(T--S);
draw(Q--I);
draw(R--M);
draw(circumcircle(A,B,C));
draw(circumcircle(P,I,N));
dot("$A$", A, dir(A));
dot("$B$", B, dir(B));
dot("$C$", C, dir(C));
dot("$I$", I, dir(315));
dot("$F$", F, dir(225));
dot("$L$", L, dir(270));
dot("$X$", X, dir(45));
dot("$N$", N, dir(60));
dot("$X'$", X1, dir(60));
dot(S);
dot(T);
dot("$P$", P, dir(270));
dot(D);
dot(E);
dot("$Q$", Q, dir(55));
dot("$M$", M, dir(225));
dot("$R$", R, dir(180));
dot("$M'$", M1, 2*dir(180));
label("$k$", K2, dir(60));
[/asy]
This post has been edited 1 time. Last edited by KST2003, May 26, 2021, 12:14 PM
Z K Y
The post below has been deleted. Click to close.
This post has been deleted. Click here to see post.
BarisKoyuncu
577 posts
#6 • 3 Y
Y by hakN, GuvercinciHoca, bin_sherlo
Here is an elementary solution.

Lemma 1: Let $P, A, I, K$ be four different points on the plane such that no three of them are collinear and $\triangle PAI\sim \triangle PIK$. Then, the $I-$symmedian in the triangle $AIK$ passes through $P$. Moreover, if $O$ is the center of $(AIK)$, then $\angle OPI=90^\circ$.
Proof
Diagram

Lemma 2: Let $ABC$ be a triangle with incenter $I$ and let $P$ be a point on $(ABC)$. The tangent at $I$ of $(AIP)$ intersects $BC$ at $K$. Then, $\triangle PAI\sim\triangle PIK$.
Proof
Diagram

Now let's get back to the problem.
Assume that the tangent at $I$ of $(AIP)$ intersects $BC$ at $K$ and $AK\cap QI=M'$.
By Lemma 2, we know that $\triangle PAI\sim \triangle PIK$ and by Lemma 1, $IP$ is a symmedian of $(AIK)$. Also, $\angle PIK=\angle PAI=\angle QAI=\angle QIA=\angle M'IA$. Since $IP$ is symmedian, we find that $M'$ is the midpoint of $[AK]$.
Since $K$ lies on $BC$, we know that $M'$ lies on the midsegment of $ABC$ that is parallel to $BC$. Also, $M'\in QI$. Hence, $M'=M$. Let $R'$ be the center of $(AIK)$. By Lemma 1, $\angle R'PI=90^\circ$. Also, clearly, $R'$ lies on $\ell$. Hence, $R'=R$. Then, $|RA|=|RK|$. Since $M$ is the midpoint of $[AK]$, we have $\angle RMA=90^\circ$, as desired.
Diagram
This post has been edited 1 time. Last edited by BarisKoyuncu, Feb 10, 2022, 12:15 PM
Reason: to add image
Z K Y
The post below has been deleted. Click to close.
This post has been deleted. Click here to see post.
mijail
121 posts
#7
Y by
Very hard problem. :love:

Let $N=AM \cap BC$ so it's enough to prove that $R$ is the circumcenter of $\triangle ANI$ because $AM=MN$. Let $P'$ the reflexion of $I$ respect $P$ then we have that $R$ is the center of $\odot (AIP')$ so it's enough to prove that $AINP'$.

Now a little motivation: If $AINP'$ is cyclic, we have that $\angle IAP = \angle AIM$ and $\angle AP'I= \angle INA$ then see that $\triangle NIA$ is almost similar to $\triangle P'AI$ then let $P''=IN \cap \odot(IAM)$ then we can guess that $\triangle NP''A \sim  \triangle P'AI$ but with angles: $$\angle NIM= \angle P''AM = \angle P'IA$$But $M$ is the midpoint of $AN$ so $AINP'$ is harmonic so $\triangle API \sim \triangle IPN$ (well-known) and this is sufficient to prove that $AINP'$ is cyclic.

That give us the idea to use phantom points, let $N'$ a point such that $\triangle API \sim \triangle IPN'$ , we don't know if this point it's on $NC$. Then the follows steps are natural:

Claim: $N' \in BC$
Proof: Let $I_A,I_B,I_C$ the excenters relative to $A,B,C$, respectively. The inversion $\Psi$ in center of $I$ that switch $A \mapsto I_A, B \mapsto I_B$ and $C \mapsto I_C$ then $P \mapsto  \Psi(P)$ and $N' \mapsto	\Psi(N')$. Then by angles: $$\angle I\Psi(P)I_A =\angle IAP = \angle PIN' =\angle \Psi(N')I\Psi(P)$$$$\angle\Psi(P')\Psi(N')I=\angle IPN'=\angle IPA =\angle II_A\Psi(P)$$This implies that $\Psi(N')II_A\Psi(P)$ is a parallelogram and we need to prove that $\Psi(N'),I,I_B,I_C$ are concyclic. By homothety with center $I_A$ with ratio $1/2$ we need to prove that the midpoints of $IP$,$I_AI_B$,$I_AI_C$,$I_AI$ are concyclic but by homothety with center $I$ and ratio $2$ the image of the midpoint of $PI$ it's on the circumcircle of $(I_AI_BI_C)$ and it's well-known that the others points also are on this circle. $\square$

Let $M' =AN'\cap IQ$ then $\angle PIN'=\angle PAI=\angle M'IA$ later see that the conjugate isogonal of $P$ respect $\triangle AIN'$ it's the $I$-Dumpty point of $\triangle AIN'$ so we have that $IM'$ it's a median this implies that $N'=N$ and $M=M'$.Then let $R= IP \cap \odot(AIN)$ so $ARNI$ is harmonic and $\angle API=\angle IPN$ then $P$ is the $R$-Humpty point of $\triangle ARN$ this implies that $IP=PR$ (well-known) so $P'=R$ and $AINP'$ is cyclic, as desired. $\blacksquare$
Z K Y
The post below has been deleted. Click to close.
This post has been deleted. Click here to see post.
NoctNight
108 posts
#8 • 1 Y
Y by archp
Let the reflection of $M$ over $\ell$ be $N$. Let $L=AI\cap \ell$. Then it suffices to prove $INPL$ cyclic as this would prove $INPLR$ cyclic giving $\angle AMR=\angle INR=\angle IPR=90$. Let the $A$-midline intersect $l$ at $X$. Then $XN, XM$ are reflections about $\ell$.

Step 1: Preparations.
$XN$ is parallel to the tangent to $(ABC)$ at $A$ because the tangent and $BC$ are anti-parallel in $\angle BAC$. Let $T=AI\cap XN$ and $D$ be the foot from $A$ onto $BC$. Then $X$ is the centre of $(AID)$ because it lies on the perpendicular bisectors of $AI$ and $AD$.

Step 2: Let $XN$ intersect $AB$ and $AC$ at $E, F$ respectively. Then $EBIL, FCIL$ cyclic.
Proof: Dilate at $A$ with factor $2$, sending $X\mapsto Y$ and $L\mapsto I$. But then $XI\perp AI$ so if $E, F\mapsto U, V$ then the lines $TUV$ and $BC$ are reflections about $TI$, so $TUV$ is tangent to the incircle of $\triangle ABC$. But then
$$\angle AIU=180-\angle AUI-\angle IAB = 180-\angle AUV-\angle IAB - \angle IUV = 180-\angle ACB-\frac{1}{2}\angle BAC - \frac12\angle BUV$$$$ = 180-\frac12\angle ACB - \frac12 \angle BAC - \frac12 (180- \angle ACB) = 90-\frac12\angle ACB - \frac12\angle BAC = \frac12 \angle ABC = \angle ABI$$so $\triangle AIU \sim\triangle ABI$ giving $AU \times AB = AI^2$. Thus dilating back, $AE \times  AB = AI \times AL$ as needed, so $EPIL$ and similarly $FCIL$ are cyclic.

Step 3: Finishing:
This gives $AN \times AP = AE \times AB = AK \times AL$ where the first equality comes from shooting lemma, so by power of a point, $INPL$ cyclic as needed.
Z K Y
The post below has been deleted. Click to close.
This post has been deleted. Click here to see post.
axolotlx7
133 posts
#9 • 1 Y
Y by GeoKing
Let $J$ be the midpoint of $AI$. We prove the following important claim:
Claim. If $AP$ meets $(PJI)$ again at $N$, then the reflection $N'$ of $N$ across $\ell$ lies on the $A$-midline.
Proof. It is easy to see by inversion at $A$ that $N'$ lies on a fixed line parallel to $BC$. Also, when $P$ is the $A$-Sharkydevil point, $\angle AJN = \angle API = 90^\circ$ implies $N'=N$, so by a homothety at $A$ with ratio $2$ it suffices to show that $W = AP \cap BC$ satisfies $\angle AIW = 90^\circ$, which is well-known. $\square$

Back to the problem, note that $R$ lies on $(PJIN)$ and $\angle NQJ = \angle JQI$; combined with the claim, this gives $M = N'$, so $\angle AMR = \angle INR = 90^\circ$.
Z K Y
The post below has been deleted. Click to close.
This post has been deleted. Click here to see post.
TestX01
339 posts
#10
Y by
:love: (i didnt invert anything!)

Subjective MOHS rating Click to reveal hidden text.

Let $A'$ be the reflection of $A$ over $M$, which lies on $BC$.

We rely on a crucial claim within this problem:

Claim: $PI$ bisects $\angle APA'$.

Suppose that $NP$ intersects $BC$ at $P'$. Instead, reverse reconstruct $A'$ such that $IPP'A$ is cyclic, and $A'$ lies on $BC$. Note that this point $A'$ is unique.

Now, by shooting lemma and Fact 5, $NI^2=NB^2=NP'\cdot NP$, thus $(IPP'A)$ is tangent to $AI$. Further, if $D$ is the foot of the angle bisector at $A$, then $ND\cdot NA=NP\cdot NP'$ as well, thus $ADP'P$ is cyclic by Power of a Point. This implies that $\angle PAI=\angle NP'B = \angle PIA'$, thus $(AIP)$ is tangent to $IA'$. Yet $(IPP'A)$ is tangent to $IA'$. This is sufficient to characterize $P$ as the $I$-Dumpty point of $\triangle AIA'$. Consequently, it is well-known that $IP$ is the $I$-symmedian in $\triangle AIA'$.

Now we claim that $A'$ indeed satisfies $PI$ bisecting $\angle APA'$. Yet check that $\angle API=\angle NIA'=\angle IPA'$ by alternate segment theorem, which provides the result.

Now, by reverse reconstruction again, if we do redefine the $M$ as the midpoint of $AA'$, and further let $IM$ intersect $AP$ at $Q$, it suffices to show that $Q$ lies on the perpendicular bisector of $AI$, or equivalently $\angle QAI=\angle AIQ$. Yet $\angle IAP = \angle A'IP=\angle QIA$ by alternate segment theorem, and symmedian property, which suffices.

Thus our claim is proved.

Now, consider the point $P$ again. Reconstruct $R$ as the center of $(AIA')$. It is well-known that $P$, being the $I$-Dumpty point of $\triangle AIA'$ satisfies $\angle IPR=90^\circ$. Yet we note that $R$ also lies on the perpendicular bisector of $AI$ by circumcenter definition. Yet this uniquely characterizes $R$.

Further, it is well-known that $ AA'PR $ is cyclic. Now, if $PI$ intersects $(AA'PR)$ again at $X$, by Fact 5, due to our claim, $X$ is the midpoint of minor arc $AA'$.

Further, $(PI, PR; PA, PA')=-1$ by right angle and bisector. Projecting this onto $(AA'PR)$, we have $RAXA'$ is harmonic, and as $X$ is midpoint of minor arc $AA'$, it is a kite, so $R,X$ are antipodes of each other. Consequently, $R$ is the midpoint of major arc $AA'$, thus as $M$ is the midpoint of $AA'$, we have $\angle AMR=90^\circ$, as desired.
Z K Y
The post below has been deleted. Click to close.
This post has been deleted. Click here to see post.
bin_sherlo
707 posts
#11
Y by
Apply $\sqrt{bc}$ inversion.
New Problem Statement: $ABC$ is a triangle with $A-$excenter $K$ and $P$ is any point on its midsegment parallel to $BC$. Let $Q$ be the foot of the altitude from $K$ to $AP$ and let $I$ be the midpoint of $AK$. Let $(AIQ)$ and $(ABC)$ meet at $M$ and $KM$ intersect $(KQA)$ at $R$. Prove that $K,I,P,R$ are concyclic.
If $AP\cap BC=V$, then $\measuredangle AKR=\measuredangle KRI\overset{?}{=} \measuredangle KPI=\measuredangle PKV$ iff $KM$ is $K-$symedian on $\triangle KAV$. Apply $\sqrt{bc}$ on $\triangle KBC$.
New Problem Statement: $ABC$ is a triangle with orthocenter $H$ and circumcenter $O$. Let $N$ be the midpoint of $AH$. An arbitrary point $D$ is taken on the circle with diameter $AO$. $(AND)$ meets the perpendicular to $AH$ at $N$ at $Q$ and $(QNH)$ intersects the ninepoint circle at $M$. Prove that $ANMD$ is a parallelogram.
Let $W$ be the point such that $ANWD$ is a parallelogram. Since the ninepoint circle is the circle shifted from $(AO)$ with length $AN$, we see that $W$ lies on the ninepoint circle. Let $D'$ be the reflection of $D$ over $NQ$. Since $A,N,D,Q$ are concyclic, by reflection over $NQ$ we get that $H,N,D',Q$ are concyclic. $QD\perp AD\parallel NW$ and $WD\perp NQ$ thus, $D$ is the orthocenter of $\triangle WNQ$. So $W,N,Q,D'$ are concyclic. These imply that $W,N,Q,D',H$ are concyclic and since $M$ is the intersection of ninepoint circle and $(HNQ)$ we get $M=W$ as desired.$\blacksquare$
Z K Y
N Quick Reply
G
H
=
a